Liouville's Theorem and constant functions

In summary, the condition that f is entire and lim z -> infinity f(z)/z = 0 implies that f is bounded.
  • #1
brian87
8
0
Homework Statement
Suppose that f is entire and [tex]\lim_{z \to \infty}\frac{f(z)}{z} = 0[/tex]. Prove that f is constant.

z, and f(z) are in the complex plane

The attempt at a solution
I've tried to find out how the condition [tex]\lim_{z \to \infty}\frac{f(z)}{z} = 0[/tex] implies the function itself is bounded, but I've not been successful in doing so.

Any hints?:confused:

Thanks :)
 
Last edited:
Physics news on Phys.org
  • #2
If a function is entire, then it has no poles for finite z. The limit condition means that f(z) is finite at infinity --- so the entire function is bounded.
 
  • #3
Hi, does this mean that the lack of singularities for finite z and it being finite at infinity imply that it's bounded on the complex plane?

Thanks

(Just looking for a clarification here)
 
  • #4
Suppose f(z) = 0 if z = 0 and log z otherwise. f is entire right? So

[tex]\lim_{z \to \infty} \frac{f(z)}{z} = \lim_{z \to \infty} \frac{f'(z)}{1} = \lim_{z \to \infty} \frac{1}{z} = 0[/tex]

by l'Hopital's rule. However f is not bounded.
 
  • #5
log z is not entire -- there is a branch cut.
 
  • #6
So genneth, is my previous statement true? Thanks
 
  • #7
There needn't be a branch cut, but then f wouldn't be a function any more. Nevermind

I'm having a hard time showing that f is bounded from the definition of the limit: for any e > 0 there is a d such that |f(z)/z -0| < e for all z satisfying |z| >= d. In other words, |f(z)| < ed for any e > 0, for all z, |z| >= d. This doesn't imply that f is bounded though. And if it does, how?
 
  • #8
Let a0+a1*z+a2*z^2+... be the power series expansion of f(z). Let g(z)=(f(z)-a0)/z. Limit of |g(z)| as z->infinity is also 0. And g(z) is bounded (use the maximum modulus principle) and entire. So g(z) is a constant. What constant?
 
  • #9
I've never heard of the maximum modulus principle. And how can g be entire if it isn't even defined for z = 0?
 
  • #10
e(ho0n3 said:
I've never heard of the maximum modulus principle. And how can g be entire if it isn't even defined for z = 0?

You MIGHT look it up. It says an analytic function takes on it's maximum modulus on the boundary of a domain, not the interior. f(z)-a0 has a power series with a common factor of z. Divide it out. g(0)=a1.
 
Last edited:
  • #11
But aren't you dividing by 0?
 
  • #12
g(z)=(a1*z+a2*z^2+...)/z=a1+a2*z+...
 
  • #13
Yes. If you do that, you're assuming z is not zero so g(0) is undefined.
 
  • #14
Fine. Then DEFINE g(0)=a1, if you wish. Or just define g(z) by the power series after z is factored out.
 
  • #15
You could have just stated that g has a removable singularity at 0.

How does the maximum modulus principle apply, i.e. what domain are you using? It certainly can't be the whole complex plane because it doesn't have a boundary.
 
  • #16
Maybe we should let the OP get a question or two in here. Since |g(z)| goes to 0 as z goes to infinity, use a circle of large enough radius that the values at |z|=R are less than epsilon.
 
  • #17
Would this solution be sufficient?

f is entire, then it has a Taylor representation about 0:

f(z) = a0 + a1*z + a2*z^2 + ...

Also, we have lim z-> infinity f(z)/z = 0, i.e. we must have a1, a2, ... = 0

thus f(z) = a0, a constant.

I used some reasoning from real analysis, can this be transferred for the complex plane though?

I still find it hard to see how the condition that lim z -> infinity f(z)/z = 0 and f is entire implies f is bounded
 
  • #18
That's REALLY unconvincing because you just stated the conclusion with no reason. It's not even true for real variables. Take f(z)=exp(-z^2). The theorem isn't true for real functions. You need to use complex methods.
 
Last edited:
  • #19
Whoops, my mistake then

But really, can someone explain to me how does the condition that f is entire and lim z -> infinity f(z)/z = 0 imply that f is bounded?

Thanks :)

[edit: It might be helpful to note that in the course I'm taking, we're doing really basic complex analysis, nothing on the maximum modulus principle, just things such as Cauchy's Theorem/Cauchy's Integral Theorem/Liouville's Theorem/Taylor, Laurent Series, classification of singular points and Cauchy's Residue Theorem]
 
Last edited:
  • #20
The maximum modulus principle follows pretty directly from Cauchy's theorem. So you can also prove that g(x)=0 using Cauchy's theorem directly. But you do need to use complex methods. If somebody else can come up with a simpler method, that would, of course, be nice.
 
Last edited:
  • #21
http://en.wikipedia.org/wiki/Liouville's_theorem_(complex_analysis)
 
  • #22
Ah, would this then work?

By Cauchy's Integral Theorem, since f is entire, we have:

f(z0) = 1/(2*pi*i) [tex]\oint\frac{f(z)}{z-z0}dz[/tex]

But since we have [tex]\lim_{z \to \infty}\frac{f(z)}{z} = 0[/tex], pick any circle with R -> infinity, thus we have f(z0) to be 0 everywhere which yields f(z) = constant
 
Last edited:
  • #23
No, it doesn't work. And it's not because of a specific flaw. It simply doesn't make sense. Look at the proof of the standard Liouville theorem and study it until you really understand it. Then tackle this problem.
 
  • #24
One thing that bugs me about that proof is that |f(u)|/|uk+1| <= M/rk+1. This is like saying: if a < b and c < d, then a/c < b/d. This is certainly not true.
 
  • #25
e(ho0n3 said:
One thing that bugs me about that proof is that |f(u)|/|uk+1| <= M/rk+1. This is like saying: if a < b and c < d, then a/c < b/d. This is certainly not true.

But |u^(k+1)|=r^(k+1) along the contour. So it's a bit more like saying a<b and c=d implies a/c<b/d. Can't argue with that one, can you?
 
  • #26
Aha! I overlooked that fact. Now it makes sense.
 
  • #27
I've looked through the proof of Liouville's theorem, it's just that I can't get that implication that such a M exists that |f(z)| < M for all z. (the earlier method i tried above did not use anything from the proof of liouville's theorem)
 
  • #28
Liouville's Theorem only applies for bounded f(z). In other words, it is assumed that f is such that |f(z)| <= M.
 
  • #29
brian87, did you look at the rest of the thread? I thought by post #16 we had practically given the solution away...
 
  • #30
Well, it seems that I got lost somewhere from post 4 onwards. (something about branch cuts)

Well, Dick, what's the part that does not make sense with the integral method?

[I note that although I know the series representation of f, this question was in a chapter before series were introduced, so I was trying to get a solution without using any series.]
 
Last edited:
  • #31
Skip over the 'branch cut' stuff. If you know f can be represented as a convergent power series, then just use it. Once you've actually figured out how the proof works then you can go back and try to write it without the series. I already said I can't tell you which part of your 'integral method' doesn't make sense. Because I can't find any parts that do make sense.
 
Last edited:

1. What is Liouville's Theorem?

Liouville's Theorem is a fundamental result in complex analysis that states any bounded entire function must be constant.

2. How is Liouville's Theorem related to constant functions?

Liouville's Theorem states that any bounded entire function, which includes constant functions, must be constant. This means that if a function is bounded and defined on the entire complex plane, it cannot have any non-constant values.

3. What is an entire function?

An entire function is a function that is defined and holomorphic (complex differentiable) on the entire complex plane.

4. Can Liouville's Theorem be applied to functions with singularities?

No, Liouville's Theorem only applies to entire functions, which do not have any singularities on the complex plane.

5. How is Liouville's Theorem used in mathematics?

Liouville's Theorem is a powerful tool in complex analysis and is used to prove other important results, such as the Fundamental Theorem of Algebra and the Maximum Modulus Theorem.

Similar threads

  • Calculus and Beyond Homework Help
Replies
10
Views
1K
  • Calculus and Beyond Homework Help
Replies
3
Views
402
  • Calculus and Beyond Homework Help
Replies
16
Views
1K
  • Calculus and Beyond Homework Help
Replies
2
Views
858
  • Calculus and Beyond Homework Help
Replies
2
Views
296
  • Calculus and Beyond Homework Help
Replies
7
Views
1K
  • Calculus and Beyond Homework Help
Replies
6
Views
746
  • Calculus and Beyond Homework Help
Replies
4
Views
963
  • Calculus and Beyond Homework Help
Replies
3
Views
1K
  • Calculus and Beyond Homework Help
Replies
2
Views
703
Back
Top